How is pressure measured? question 1 options: pressure is measured as force per inch. pressure is measured as force per gram. pressure is measured as force per unit area. pressure is measured as force divided by mass.

Answers

Answer 1

Pressure is measured as force per unit area, which is the third option.

What is pressure?

Pressure is amount of force that is applied over a given area divided by the size of this area.

Pressure is calculated by multiplying the force applied on the object by the area covered.

Since force is measured in Newtons (N) and area is measured in m², the unit of pressure is Nm².

Therefore, pressure is measured as force per unit area.

Learn more about pressure at:

https://brainly.com/question/12971272

#SPJ1


Related Questions

If f is the magnitude of the force on the test charge due to only one of the other charges, what is the magnitude of the net force acting on the test charge due to both of these charges?.

Answers

The net force acting on test charge is resultant of two force i.e.,  √2 F

The magnitude of force on test charge due to one charge Q is:

                F = k qQ / r²

where k = 9 × 10^9

Similarly the force due to second charge will be:

         F = k qQ / r²

Now both the charges of magnitude Q is present and it is pointed at right angle to each other then the force  will be resultant i.e.,

                    F = √ F² + F² = F√2

 Hence, net force acting on test charge is √2 F.

  Learn more about charges here:

        https://brainly.com/question/15840431

             #SPJ4

A glass optical fiber is used to transport a light ray across a long distance. The fiber has an index of refraction of 1.510 and is submerged in water, which has an index of refraction of 1.333. What is the critical angle (in degrees) for the light ray to remain inside the fiber

Answers

The answer is 73.13°.

According to snell's law,

n1sinθi = n2sinθr

n1/n2 = sinθr/sinθi

The critical angle is the angle of incidence at the denser medium when the angle of incidence at the less dense medium is 90°

This means i=C and r = 90°

The Snell's law formula will become

n1/n2 = sinC/sin90°

n2/n1 = 1/sinC

Where n1 is the refractive index of the less dense medium = 1.473

n2 is the refractive index of the denser medium = 1.540

Substituting the values in the formula,

1.540/1.473 = 1/sinC

1.045 = 1/sinC

SinC = 1/1.045

SinC = 0.957

C = sin^-1(0.957)

C = 73.13°

The refractive index of glass is 1.5. It way that the velocity of mild in glass is 1.5 times slower than the rate of mild in a vacuum, the speed of light in glass isn't always unbiased of the shade of mild.

Refractive index is likewise the same as the velocity of light c of a given wavelength in an empty area divided with the aid of its speed v in a substance or n = c/v.

The index of refraction of fabric is a ratio that compares the velocity of light in a vacuum ( c=3.00x108ms ) to the velocity of mild in that precise medium. Because the index of refraction increases, the amount that the material bends the mild increases.

Learn more about the index of refraction here https://brainly.com/question/12469161

#SPJ4

The critical angle (in degrees) for the light ray to remain inside the fiber is 73.13°.

According to snell's law,

n1sinθi = n2sinθr

n1/n2 = sinθr/sinθi

The critical angle is the angle of incidence at the denser medium when the angle of incidence at the less dense medium is 90°.

This means i=C and r = 90°

The Snell's law formula will become

n1/n2 = sinC/sin90°

n2/n1 = 1/sinC

Where n1 is the refractive index of the less dense medium = 1.473

n2 is the refractive index of the denser medium = 1.540

Substituting the values in the formula,

1.540/1.473 = 1/sinC

1.045 = 1/sinC

SinC = 1/1.045

SinC = 0.957

C = sin^-1(0.957)

C = 73.13°

Learn more about critical angle here https://brainly.com/question/15009181

#SPJ4

While standing at the top of an 100 m high observatory, you accidentally dropped your phone through the grates (it is now falling straight down to the ground). a. What is the velocity of the phone after 4.0 s? b. How far does the phone fall during this time? C. Will your phone hit the ground? If not, how long more before it hits the ground?​

Answers

Using the idea of motion under gravity, it will take 0.5 seconds more before the phone hits the ground.

What is the distance covered?

The distance covered is obtained form the equations of kinematics under gravity.

Now;

v = u + gt

Recall that it was dropped from a height hence u = 0 m/s

v = gt

v = 9.8 m/s^2 * 4 s

v = 39.2 m/s

Now;

h = ut + 1/2gt^2

but u = 0 m/s

h = 1/2gt^2

h = 1/2 * 9.8 * (4)^2

h = 78.4 m

The phone  will not hit the ground within this time

For the phone to hit the ground;

h = 1/2gt^2

if h = 100 m

100 = 1/2 * 9.8 * t^2

2 * 100/9.8 =  t^2

t = √2 * 100/9.8

t = 4.5 seconds

It will take about 0.5 seconds more before the phone will hit the ground.

Learn more about motion under gravity:https://brainly.com/question/15120445

#SPJ1

Answer the following questions.
1. What is the velocity of a car that traveled a total of 75 kilometers north in 1.5 hours?
2. What is the velocity of a plane that traveled 3,000 miles from New York to California in 5.0 hours?
3. John took 45 minutes to bicycle to his grandmother's house, a total of four kilometers. What was his speed in km/hr?.
4. It took 3.5 hours for a train to travel the distance between two cities at a speed of 120 miles/hr. How many miles lie between the two cities?​

Answers

3.if time=45mins,D=4km,2700secs,4000m then. S=d/t S=1.4m/s

. A man of mass 80 kg stands in a lift, what is the reaction from the floor of the lift if the lift;A. moves upwards with steady speed. moves upwards with acceleration 0.5m/sC. moves downward with acceleration 0.4m/s(g - 9.8m/s')​

Answers

The reaction of the floor is as follows:

R = 784 NR = 816 N

What is the reaction from the floor for a man in a lift?

The reaction of the floor on the man is equal to the weight of the man and the upward force.

Reaction, R = mg + ma

A)  When the lift moves upwards with steady speed.

R = 80 * 9.8 + * 80 * 0

R = 784 N

B) When the lift moves upwards with acceleration 0.4m/s

R = 80 * 9.8 + * 80 * 0.4

R = 816 N

In conclusion, the reaction of the floor changes as the body accelerates upwards.

Learn more about reaction and weight at: https://brainly.com/question/25555136

#SPJ1

4. An applied force of 6.2 N acts on a 2.1-kg object, pushing it horizontally across a surface
where the coefficient of kinetic friction is 0.15.
a. Draw a free body diagram for the body
b. Determine the frictional force acting.
c. Determine the net force acting on the body
d. Determine the object's acceleration.
Step by step, thank you
[2C]
[2A]
[2A]
[2A]

Answers

The frictional force on the body is  0.25 N the net force on the body is  5.95 N while the acceleration produced is 2.83 m/s^2

What is the net force?

The net force is used to describe the resultant force that acts on the object.

The frictional force that acts on the object s obtained from;

Ff = 0.15 * 2.1-kg * 9.8 m/s^2

Ff = 0.25 N

The net force that acts on the body = F - Ff where F is the applied the force

=  6.2 N - 0.25 N

Net force = 5.95 N

Now;

Net force = ma

a= Net force /m

a = 5.95 N/2.1-kg

a = 2.83 m/s^2

Learn more about the net force:https://brainly.com/question/18031889

#SPJ1

A speed skater moving across frictionless ice at 8.0 m/s hits a 5.0-m-wide patch of rough ice. She slows steadily, then continues on at 6.0 m/s. What is her acceleration on the rough patch

Answers

Her acceleration on the rough patch is 2.8m/s².

To find the answer, we need to know about the Newton's equation of motion.

What are the Newton's equation of motion?

These are as follows

V= U +atS = Ut+1/2 at²V²-U²= 2aS

V= Final velocity

U = initial velocity

t= time

S= distance

What will be acceleration if initial velocity, final velocity and distance are 8 m/s , 6 m/s and 5m respectively?Here U= 8 m/s, V = 6 m/s and S = 5mSo, 6²-8²= 2a× 5=10a

36-64 = 10a

-28= 10 a

a= -28/10= -2.8 m/s²

Negative sign indicates the decrease of acceleration.

Thus, we can conclude that the acceleration on the patch is 2.8 m/s².

Learn more about the Newton's equation of motion here:

brainly.com/question/8898885

#SPJ4

A vehicle starts from rest and accelerates uniformly for 12 seconds to a
velocity of 10ms1. It then runs at a constant velocity and is finally came to rest
in 66m with constant . The total distance covered by the vehicle is
580m. Find the value of acceleration and the time taken.

Answers

The value of the acceleration is 0.76 m/s² and the total time taken by the vehicle is 39 seconds.

Acceleration of the vehicle

The acceleration of the vehicle before coming to rest is calculated as follows;

v² = u² + 2as

where;

v is the final velocityu is the initial velocitya is the accelerations is the distance traveled before stopping

the car came to rest with constant velocity attained after 12 seconds.

the initial velocity of the car before 12 seconds is zero.

v² = 0 + 2as

a = v²/2s

a = (10²)/(2 x 66)

a = 0.76 m/s²

Time of motion of the vehicle

d = ut + ¹/₂at²

where;

d is the total distance traveledt is the time of motiona is accelerationu is initial velocity of the vehicle

580 = 0 + ¹/₂(0.76)t²

580 = 0.38t²

t² = 580/0.38

t² = 1,526.3

t = √1,526.3

t = 39 seconds

Thus, the value of the acceleration is 0.76 m/s² and the total time taken by the vehicle is 39 seconds.

Learn more about time of motion here: https://brainly.com/question/2364404

#SPJ1

At which points on the roller coaster is the car not moving?

A
A and E

B
B and C

C
C and D

D
D and E

Answers

The maximum potential energy of the car or the when car is not moving at points A and E; option A.

What is potential energy?

Potential energy is the energy of a body due to its state or position.

In the rollercoaster motion shown, the maximum potential energy occurs when the car is no longer moving.

At different points other than at maximum potential energy, the energy is a combination of potential and kinetic energy.

The maximum potential energy occurs at A and E, at which point the car is not moving.

In conclusion, at maximum potential energy, the car is not moving.

Learn more about potential energy at: https://brainly.com/question/14427111

#SPJ1

Calculate the frequency of the 3rd harmonic of a 0.5 meter string with a linear density of 0.03 kg/m at a tension of 50 newtons.

Answers

Frequency of third harmonic is 122.48 HZ

Theory:

Frequency of third harmonic with length L , linear density μ , Tension

T is given as

                  fₙ= n v/2L

                   fₙ = n/2L × √T/√μ

      plugging in the values here-

                   fₙ =  3/ 2×0.5 × √50 / √0.03

                   fₙ = 122.48  Hz

          Thus, the frequency of third harmonic is 122.48 HZ

        Learn more about frequency here:

               https://brainly.com/question/14888403

                      #SPJ4    

If the radio waves transmitted by a radio station have a frequency of 79.5 MHz, what is the wavelength of the waves, in meters

Answers

If the radio waves transmitted by a radio station have a frequency of 79.5 MHz, the wavelength of the waves, in meters is 3.77m.

What are radio waves?The electromagnetic spectrum's longest wavelengths, which are found in radio waves, are normally found at frequencies of 300 gigahertz. They may be reflected and refracted to change direction, and they do not harm the human body if they are absorbed by it. They are perfect for communicating due to their characteristics.

What are Frequency and wavelength?When describing the temporal rate of change seen in oscillatory and periodic phenomena like mechanical vibrations, radio waves, and light, frequency is a crucial parameter utilized in science and engineering.The length of a waveform signal that is propagating in space or over a wire is measured by the separation between identical points (adjacent crests) in the adjacent cycles. Frequency, or the number of wave cycles per second, is inversely related to wavelength.

Given: Velocity of light,  c = 3.00 x 10⁸ m/s  m/s

Frequency, f = 7.95 x 10⁷/s

For wavelength, the required formula is

[tex]λ = c/ f \\λ = 3 * 10 ^{8} / 7.95 * 10^{7} \\λ = 3.77[/tex]

Hence, the required wavelength is 3.77 meters.

To learn more about frequency and wavelength refer to:

https://brainly.com/question/12924624

#SPJ4

An aircraft executes a horizontal loop of radius 1.00 km with a steady speed of 900 km/h.Compare it's centripetal acceleration with the acceleration due to gravity.​

Answers

Answer:

900 km/hr = 9E5 m / 3600 sec = 250 m/sec

ah = v^2 / R = 250^2 m/s / 1000 m = 62.5 m/s^2   horizontal acceleration

av = 9.8 m/s^2      vertical acceleration (due to gravity)

ah / av = 62.5 / 9.8 = 6.38   centripetal acceleration is greater

Without using afood flask, suggest another ways would you keep food warm a till to the hospital

Answers

Another way to keep food warm till you arrive at the hospital is by putting the food in a foil and wrapping it up with a towel to retain the heat.

What is Food preservation?

Food preservation is defined as the process by which a cooked food or process food is kept in such a way that microorganisms cannot affect their taste and texture.

To prevent a cooked food from getting cold, you can put the food in an aluminium foil and wrap it with a towel to prevent heat loss.

Learn more about food preservation here:

https://brainly.com/question/16972821

#SPJ1

In which of the following scenarios is the total momentum of the system conserved?

Answers

Answer:

The total momentum of a system is conserved only when the system is closed.

Explanation:

If you place a 3.5-kg object on a spring which has a spring constant 144-N/m, and stretch the spring so the object starts oscillating. How much time does it take to complete one cycle

Answers

Answer:

M = 3.5 kg         mass of object

F = - K x     where K is the spring constant

K = 144 N/m

ω = (K/M)^1/2     for simple harmonic motiom

ω = (144 kg/s^2 / 3.5 kg)^1/2 = 6.41 / sec

f =  ω / (2 * π)  = 1 / P

P = (2 Π / ω) = 2 * 3.14 / 6.41 sec

P = .98 sec

Check: f = 1 / P = 1.02 /sec

ω = 2 * π * f = 2 * 3.14 * 1.02 = 6.41 sec

A block of mass 60kg measures 6cm by 5cm by 4cm. Calculate i) The maximum pressure it can exert. The minimum pressure. ii) A block of mass 60kg measures 6cm by 5cm by 4cm . Calculate i ) The maximum pressure it can exert . The minimum pressure . ii )​

Answers

Answer:

4.9 is the answer

Explanation:

pressure is force divided by area

force = mass x acceleration due to gravity

= 60kg x 9.8 m/s

area = length x width x height

=6 x 5 x 4

= 120

pressure = 588/120

= 4.9 pascal

The correct answer for the value of (i) Maximum pressure is [tex]24.5 \dfrac{N}{cm^2}[/tex](ii)Minimum pressure is [tex]19.6 \dfrac{N}{cm^2}[/tex].

Given:

Weight of the block:  [tex]60[/tex] kg

Dimensions of the box:  [tex]6 *4* 5[/tex]

Value of gravitational constant:  [tex]9.8[/tex] m/s²

Now,

[tex]Pressure = \dfrac{Force}{Area}[/tex]

[tex]Force = mass*accelaration[/tex]

[tex]= 60 * 9.8[/tex]

[tex]= 588 N[/tex]

Minimum pressure is when area is maximum:

[tex]= \dfrac{588}{6*5}\\\\= 19.6 \dfrac{N}{cm^2}[/tex]

Maximum pressure when area is minimum:

[tex]= \dfrac{588}{6*4} \\\\= 24.5 \dfrac{N}{cm^2}[/tex]

Maximum and minimum pressure are   [tex]24.5 \dfrac{N}{cm^2}[/tex]  and  [tex]19.6 \dfrac{N}{cm^2}[/tex] respectively.

Learn more about Pressure here:

https://brainly.com/question/21173163

#SPJ7

You look in the sky and see two jetliners that you know are of equal size, yet one appears to be much larger. Because of your knowledge of __________, you will assume that the smaller jetliner is farther away

Answers

Answer:

Because of the knowledge of relative size, it will be assumed that the smaller jetliner is farther away.

Explanation:

According to the theory of relative size, the distance that an object has to the viewing individual affects the perception of the individual regarding the size of the object.

As stated in this case, one of the jetliners is farther away from the other. Therefore, even if the jets are of equal size, the one that is at a greater distance is perceived to be smaller as it is at a greater viewing range. The one that is nearer to the individual seems bigger in comparison to the one farther away due to a closer viewing range.

Therefore, the jet that is nearer appears larger.

To know more about relative size, refer to:

https://brainly.com/question/19998265

#SPJ4

A 500 N weight is hung at the middle of a rope attached to two buildings at the same level. If the breaks in the tension exceed 1800 N, what is the minimum angle the rope can make with the horizontal? Group of answer choices 4o 8o 11o 18o

Answers

Answer:

The correct choice is 8o.

Explanation:

The given weight is hanging at the mid point of the rope and the buildings are at the same level, Obtain the given equation by equating the vertical components of force,

2Tsin(φ)=W

where φ is the angle made by rope with the horizontal.

Given T=1800 N and W=500 N, the value of φ is calculated as follows,

2*1800*sin(φ)=500

sin(φ)=0.1388

φ=7.90

φ≈8o

Check out similar questions.

brainly.com/question/13735944

#SPJ10

Two objects, m1 = 0.6 kg and m2 = 4.4 kg undergo a one-dimensional head-on collision
Their initial velocities along the one-dimension path are vi1 = 32.4 m/s [right] and vi2 = 8.6 m/s [left].
The two objects stick together after the perfectly inelastic collision
a) Calculate the velocity after the collision.
b) Determine how much kinetic energy is lost due to the collision

Answers

Explanation:

Hello !

look at the attachment above ☝️ and if you have any questions your welcome.

a) The velocity after the collision.is 11.456 m/s.

b) The kinetic energy lost due to the collision is 44.564 J.

What is conservation of momentum principle?

When two bodies of different masses move together each other and have head on collision, they travel to same or different direction after collision.

The external force is not acting here, so the initial momentum is equal to the final momentum. For inelastic collision, final velocity is the common velocity for both the bodies.

m₁u₁ +m₂u₂ =(m₁ +m₂) v

Given are the two objects, m1 = 0.6 kg and m2 = 4.4 kg undergo a one-dimensional head-on collision. Their initial velocities along the one-dimension path are vi1 = 32.4 m/s [right] and vi2 = 8.6 m/s [left].

(a) Substitute the values, then the final velocity will be

0.6 x32.4 +4.4 x 8.6 = (0.6+4.4)v

v = 11.456 m/s

Thus, the velocity after collision is 11.456 m/s.

(b) Kinetic energy lost due to collision will be the difference between the kinetic energy before and after collision.

= [1/2m₁u₁² +1/2m₂u₂² ] - [1/2(m₁ +m₂) v²]

Substitute the value, we have

= [1/2 x 0.6 x32.4² + 1/2 x4.4 x 8.6²] - [1/2 x(0.6+4.4)11.456²]

= 44.564 J

Thus, the kinetic energy lost due to the collision is 44.564 J.

Learn more about conservation of momentum principle

https://brainly.com/question/14033058

#SPJ2

a ray of light incident on a mirror, at an angle of 45°. Another mirror is placed at an angle of 45° to the first ones as shown. Sketch the patch of the ray until it emerges from the two mirrors (2mks) 45° www.www 45° ​

Answers

Answer:

If the ray of light is deflected by 45 degrees by the first mirror its total deflection by mirror (I) is 90 deg. (incident = 45 and exit ray equals 45 deg)

The second mirror will cause a net deflection of 90 degrees and the total deflection will be 180 deg or in opposite  direction to the  incident ray.

Why should a scientist not be concerned when a fellow scientist responds to their conclusions with skepticism?

Answers

ABCDEDJGHIKLMNOP

just wanted to see if bold work

A brightfield microscope equipped with a ____________ ___________ stop is able to block light from the illuminator creating bright objects on a dark background.

Answers

Answer: A Brightfield microscope equipped with an opaque light stop is able to block light from the illuminator and creating bright objects on a dark background.

Explanation: To find the correct statement about Brightfield microscope, we need to know more about the Brightfield microscope.

what is Brightfield microscope?The Brightfield microscopic technique is generally used with the compound microscopes and, these are one, among the widely used optical illumination techniques. The Brightfield microscope or Compound light microscope is an optical instrument, which uses light rays and produces dark images with a bright background.How Bright field microscope works?A brightfield microscope equipped with a opaque light stop, will produce bright objects on a dark background by blocking the light illuminated from the light source. This opaque light stop, blocks the light rays that arising from the illuminator to the objective lenses and, only allows the reflected or transmitted light of the specimen. As a result, we get bright objects with a dark background.

Thus, we can conclude that, A Brightfield microscope equipped with an opaque light stop is able to block light from the illuminator and creating bright objects on a dark background.

Learn more about the Brightfield microscope here: https://brainly.com/question/28019837

#SPJ4

The density of a certain type of plastic is 0.77 g/cm3. If a sheet of this plastic is 10.0 m long, 1.0 m wide, and 1 cm thick, what is its mass

Answers

Mass is 77,000 g

Given

The density of a certain type of plastic = 0.77 [tex]$\mathrm{g} / \mathrm{cm}^{3}$[/tex]

Length = 10.0 m = 1000 cm

Width = 1 m = 100 cm

Thickness = 1 cm

Formula

Volume = length x width x thickness

Mass = Density x Volume

Explanation

Density

Density is how much "stuff" is contained in a specific quantity of space is determined by its density. For instance, a block of the harder, lighter element gold (Au) will be denser than a block of the heavier element lead (Pb) (Au). Styrofoam blocks are less dense than bricks. Mass per unit volume serves as its definition.

Volume V = 1000 x 100 x 1

             V = 100,000 [tex]cm^{2}[/tex]

Mass M= 0.77 [tex]$\mathrm{g} / \mathrm{cm}^{3}$[/tex] x 100,000 [tex]cm^{2}[/tex]

M = 77,000 g

So, Mass is 77,000 g

Learn more about this problem here:

https://brainly.com/question/598902

#SPJ4


Which statement is true of a piece of ice at 0°C that is put into a freezer at
-18°C?

Answers

The statement which is true about a piece of ice at 0°C which is put into a freezer at -18°C is it having the temperature of the freezer.

What is Temperature?

This is referred to the degree of hotness or coldness of a body and the unit is Celsius or Kelvin.

The ice at 0°C will experience a change in temperature of the freezer when put in it in this scenario.

Read more about Ice here https://brainly.com/question/2267329

#SPJ1

Throw two balls from the same height at the same time at an initial speed of 20 m/s. One is thrown vertically down, while the other is thrown vertically up. We want to find the time difference between their landing.

Answers

The time difference between their landing is 2.04 seconds.

Time of difference of the two balls

The ball thrown vertical upwards will take double of the time taken by the ball thrown vertically downwards.

Time difference, = 2t - t = t

t = √(2h/g)

where;

h is the height of fallg is acceleration due to gravity

Apply the principle of conservation of energy;

¹/₂mv² = mgh

h = v²/2g

where;

v is speed of the ball

h = (20²)/(2 x 9.8)

h = 20.41 m

Time of motion

t = √(2 x 20.41 / 9.8)

t = 2.04 s

Thus, the time difference between their landing is 2.04 seconds.

Learn more about time of motion here: https://brainly.com/question/2364404

#SPJ1

_____________is the addition of wave energy as waves interact, producing larger waves and ____________ is the subtraction of wave energy as waves interact, producing smaller waves.

Answers

Constructive interference is the addition of wave energy as waves interact, producing larger waves and destructive interference is the subtraction of wave energy as waves interact, producing smaller waves.

To find the answer, we need to know about the interference of waves.

What's the interference of waves?Interference of waves is the result of superposition of waves (transverse or longitudinal) at a certain place. Interference is of two typesConstructive interferenceConstructive interferenceDestructive interferenceHow are the constructive and destructive interference formed?When two waves are superimposed within the phase, then constructive interference pattern is formed.When two waves are superimposed with out of phase, destructive interference pattern is found.

Thus, we can conclude that the constructive interference is the addition of wave energy and destructive interference is the subtraction of wave energy.

Learn more about the interference of waves here:

brainly.com/question/12466679

#SPJ4

(picture attached) use grass method
4. A block of unknown substance is submerged in water. A light ray in the water strikes the angle of 16°, what is the index of refraction of the unknown substance? (nwater = 1.33) substance at an angle of 45° from the normal. If the refracted ray in the substance is at an 5. If a light ray passes from a substance with low index of refraction to another substance with high index of refraction, will the ray bend away from or closer to the normal? Why? 6. What is the angle of refraction? n₁=1 24° n₂ = 1.8​

Answers

Answer:

Explanation:

Step 1

1 of 2

\textbf{Given}Given

n_{w}=1.33n

w

=1.33

$\theta_w=31\text{ ^\circ

}$

$\theta_b=27\text{ ^\circ

}$

\textbf{Approach}Approach

In this problem we are going to use Snell's law.

\textbf{Solution}Solution

The definition of Snell's law of refraction is

\begin{align} {n_1\cdot\sin \theta_1}={n_2\cdot \sin \theta_2} \end{align}

n

1

⋅sinθ

1

=n

2

⋅sinθ

2

 

where indexes 11 and 22 represent two different mediums. Since the motion is in the water we write

\begin{align} &{n_{w}\cdot\sin\theta_{w}}={n_{b}\cdot\sin\theta_{b}} \\ &{n_b}={n_{w}\cdot \frac{\sin\theta_{w}}{ \sin \theta_{b}} } \\ &{n_b}={1.33\cdot \frac{\sin31^\circ}{ \sin27^\circ} } \\ &\boxed{{n_b}=1.5} \end{align}

 

n

w

⋅sinθ

w

=n

b

⋅sinθ

b

n

b

=n

w

sinθ

b

sinθ

w

n

b

=1.33⋅

sin27

sin31

n

b

=1.5

If an electron has an uncertainty in its velocity of 1.40 m/s, what is the uncertainty (in meters) in its position

Answers

Uncertainty in position:

If the electron has an uncertainty in its velocity of 1.40 m/s then the uncertainty in its position is [tex]4.14\times10^{-4} \text{ m}[/tex].

Heisenberg's uncertainty principle to calculate the required:

Step-1:

We have to apply Heisenberg's uncertainty principle to calculate the uncertainty in the position of an electron. According to the principle:

[tex]$\Delta x \Delta p \geq \frac{h}{4 \pi}$[/tex]

Here,

[tex]$\Delta x$[/tex] is the uncertainty on the position measurement

[tex]$\Delta p$[/tex] is the uncertainty on the momentum measurement

h is the Planck constant.

It is known that the momentum of a particle is calculated as, the product of the mass of the particle and its velocity.

Therefore,

p=mv

Thus the Heisenberg principle becomes:

[tex]$m \Delta x \Delta v \geq \frac{h}{4 \pi}$[/tex]

Here [tex]$\Delta v$[/tex] is the uncertainty in the velocity measurement.

It is given that, [tex]$\Delta v$[/tex]=1.40 m/s. The particle is electron here, thus the mass of the particle m=[tex]9.1 \cdot 10^{-31} \mathrm{~kg}[/tex] and the value of the Plank's constant is, h=[tex]6.62 \times 10^{-34} \mathrm{~m}^{2} \mathrm{~kg} / \mathrm{s}[/tex]

Step-2:

Substituting the values into the equation to get the value of the position uncertainty.

[tex]\Delta x \geq \frac{h}{4 \pi\times m\times \Delta v}\\\geq\frac{6.62\times10^{-34}}{4\pi \times 9.1\times10^{-31}\times1.40} \text{ m}\\\geq 4.14\times10^{-4} \text{ m}[/tex]

To know more about the uncertainty in position, refer to:

https://brainly.com/question/9574825

#SPJ4

A ball is launched from the ground with a horizontal speed of 30 m/s and a vertical speed of 30 m/s. How long will it take to get to the top of its trajectory?

Answers

The ball will take 3 seconds to get to the top of its trajectory When it is launched from the ground with a horizontal speed of 30 m/s and a vertical speed of 30 m/s

What is Trajectory ?

When any object is thrown from horizontal at an angle θ except 90°, then the path followed by it is called trajectory, the object is called projectile and its motion is called projectile motion.

Here, we have a type of motion called projectile motion and it is pretty similar to an upside down parabola. The top of the trajectory is the vertex of the parabola and is also when v = 0.

Given :

Horizontal speed= 30m/sVertical Speed= 30 m/s

This problem is now relatively easy because we only need to find the vertical distance so we can ignore horizontal speed and use

vy = vy0 + ayt

Plug in our givens

0 = 30  - 10t

solve for t

t = 3 seconds

Hence, The ball will take 3 seconds to get to the top of its trajectory When it is launched from the ground with a horizontal speed of 30 m/s and a vertical speed of 30 m/s

Learn more about Projectile motion here ;

https://brainly.com/question/11049671

#SPJ1

What is the current flowing through the circuit shown? (V= 120 V, R₁ = 20 02,
R₂= 50 Q, R3= 1022) (Ohm's law: V = IR)
R₁
ww
ww
R₂
ww
R₂

Answers

Assumed that resistors are connected in series .

R_net=20+50+10=80ohm

Voltage=V=120V

Current=I

V/R120/803/21.5A
Other Questions
Write a letter to congressman about climate change The sum of 2 numbers is 24. one number is 3 times great as the other. what are the 2 numbers? A firm's attempts to shorten the length of time a process takes, may lead to disappointing outcomes because of ______. Using Sarais measurements, Line 3 must be parallel to Line 5 according to which postulate or theorem?I need help ASAP please!! 2. The force of attraction between the particles is greater in ice than in water. True or false Sensory impulses traveling through the posterior root will reach the _______ upon entering the spinal cord. How many characteristics of a population does a random variable represent?1. As many as you wish2. Single3. Less than one4. Two or more Find the length of CD shown in red below. Show all work.PLEASE HELP 1. Choose three gods and goddess from the list. What are their names? Describe the depiction of each god or goddess that you chose. What is each known for? Which of the following is an example of a service industry job in an urban center?a farmera bank clerka factory workera railroad worker What issue was a focus of the Democratic-Republican Party? 5Type the correct answer in the box. Use numerals Instead of words. If necessary, use / for the fraction bar.Mark wants to fence 4 rectangular gardens, each with a length of 9 feet and a width of 4 feet. What is the total length of fencing Mark needs tosurround all 4 gardens?Mark needs 110feet of fencing. Research found that mothers and fathers showed the same level of sensitivity to sons, but fathers were ________ sensitive to daughters than mothers. more less equally none of the choices An electroencephalograph shows that during sleep we pass through a cycle of stages, each with characteristic brain waves. As the night progresses, the REM stage: ATwo similar triangles are shown.RSTSMark this and returnNXYARST wasAZXY.O rotatedO reflectedO translatedO dilatedNerthen dilated, to createSubmit Suppose a group of researchers wants to investigate the relationship between daily mindfulness meditation and cortisol (stress hormone) levels in the body over a 20-year time period. Which research design would allow them to measure changes with age and within individuals over time, even though it would be expensive, limited to one cohort, take a long time, and they might have people drop out of the study f the coordinates of point A are (8 , 0) and the coordinates of point B are (3 , 7), the y-intercept of is . If the coordinates of point D are (5 , 5), the equation of line is y = x + . Two infinite sheets of current flow parallel to the y-z plane. The left-hand sheet, which intersects the x-axis at x = 0, consists of an infinite array of wires parallel to the z-axis with a density n = 910 wires/m and a current per wire of IL = 0.17 A in the +z direction. The right-hand sheet, which intersects the x-axis at x = a = 12 cm, is identical to the left-hand sheet, except that it has a current per wire of IR = 0.17 A in the -z direction.a) Calculate the y-components of the net magnetic field in the following places: x1 = -15 cm, x2 = 6 cm, and x3 = 24 cm. (The x- and z-components of the B-field are zero.)B(x1)y = TB(x2)y = TB(x3)y = Tb) Suppose the above configuration of currents is unchanged except that the direction of the current IR is reversed so that now IR also flows in the +z direction. (The magnitude remains the same.) Calculate the y-components of the net magnetic field at the same positions as in part a).B(x1)y = TB(x2)y = TB(x3)y = Tc) Return to the configuration of part a). Suppose you want to have the region 0 < x < a able to confine electrons (qe- = -1.60 x 10-19 C, me- = 9.11 x 10-31 kg) that have been accelerated from rest through a 66 V electrostatic potential. If the electrons are to be stacked in circular orbits parallel to the x-z plane with centers on the plane x = a/2, what is the minimum current per wire required if IL and IR are equal in magnitude but opposite in direction?IL = A 7. 1 bag of beans weighs 0.7 lb. What is the weight in pounds of 2.5 bags of beans? I uploaded the chart for the questionWhat is the amount of dividend paid in 2020? a. $163 b. $231 c. $187 d. $2480.5 points QUESTION 3What is the net income in 2021? a. $1,155 b. $3,288 c. $3,724 d. $1,2401 points QUESTION 4What is the total assets in 2020? a. $7,119 b. $6,843 c. $9,739 d. $6,5510.5 points QUESTION 5What is the owners' equity in 2021? a. $9,281 b. $4,968 c. $6,569 d. $3,7261 points QUESTION 6What is the cash flow from assets in 2021? a. $4,332 b. $1,578 c. $6,712 d. -$2,3801 points QUESTION 7What is the cash flow to creditors in 2021? a. $4,332 b. $6,712 c. $1,580 d. -$2,3801 points QUESTION 8What is the cash flow to stockholders in 2021? a. $6,712 b. -$2 c. $4,332 d. -$2,3800.5 points QUESTION 9M&M Foods' current ratio in 2021 is _____. a. 6.77 b. 1.45 c. 3.31 d. 2.851 points QUESTION 10M&M Foods' quick ratio in 2021 is _____. a. 2.68 b. 2.85 c. 5.33 d. 1.401 points QUESTION 11M&M Foods' day's sales in receivables in 2021 is _____. a. 48 days b. 57 days c. 74 days d. 66 days1 points QUESTION 12M&M Foods' total asset turnover in 2021 is _____. a. 0.72 b. 0.90 c. 2.03 d. 1.851 points QUESTION 13M&M Foods' day's sales in inventory in 2021 is _____. a. 102 days b. 159 days c. 83 days d. 135 days1 points QUESTION 14M&M Foods' profit margin in 2021 is _____. a. 38 b. 19 c. 0.19 d. 0.380.5 points QUESTION 15M&M Foods' return on assets in 2021 is _____. a. 0.48 b. 48 c. 17 d. 0.171 points QUESTION 16M&M Foods' return on equity in 2021 is _____. a. 0.25 b. 19 c. 25 d. 0.191 points QUESTION 17M&M Foods' total debt ratio in 2021 is _____. a. 0.15 b. 0.30 c. 15 d. 300.5 points QUESTION 18M&M Foods' equity multiplier in 2021 is _____. a. 1.43 b. 1.17% c. 1.17 d. 1.43%1 points QUESTION 19M&M Foods' times interest earned ratio in 2021 is _____. a. 7.83% b. 7.83 c. 0.13 d. 131 points QUESTION 20If M&M Foods has 1,000 shares outstanding and the stock is sold for $25 per share on 12/31/2021, its PE ratio is _____. a. 31% b. 31 c. 20.16 d. 20.16%1 points QUESTION 21If M&M Foods has 1,000 shares outstanding and the stock is sold for $25 per share on 12/31/2021, its market-to-book ratio is _____. a. 5.03 b. 4.12% c. 4.12 d. 5.03%1 points QUESTION 22M&M Foods' internal growth rate in 2021 is _____. a. 0.16 b. 0.54 c. 0.25 d. 0.801 points QUESTION 23M&M Foods' sustainable growth rate in 2021 is _____. a. 0.25 b. 0.54 c. 0.16 d. 0.80